What is your favored interpretation of quantum mechanics?

In summary: Relative stateIn my opinion it is the best interpretation so far. It avoids the problems of the other interpretations and moreover it is empirically testable. It does suffer from some problems though. For instance it does not really explain how a single value of the measured observable is picked up.- Information theoreticI cannot imagine anything more radical. It is basically the 'last thing to be changed'. It requires also the necessary introduction of macroscopic concepts.- Quantum logicI cannot imagine anything more radical. It is basically the 'last thing to be changed'. It requires also the necessary introduction of macroscopic concepts.- Relational interpretationToo solipsistic. It is basically saying
  • #36
Cane_Toad said:
We're back to the original axiom, QM works, ergo, deal with it.

Yes, but now we know where the axioms come from to begin with. In fact quantum mechanics is now the easy part. We know where the path integral formulation comes from, and from that we can get the Schrodinger equation and Hilbert spaces, etc. This at least has a basis in logic and is thus now more intuitive. What is not yet derived from logic alone is the particular Lagrangians of common physical situations. And I don't know why we have 3+1 spacetime dimensions. Though Feynman claims to have derived gravity from QM. I have yet to look into this.
 
Physics news on Phys.org
  • #37
A first question on a quick reading...

In a sample space, the probability of a conjunction results from multiplying the probabilities of each event

Why do you assume samples be statistically indepdendent?

/Fredrik
 
  • #38
nabuco said:
Well, I don't claim I understand what is going on, I just think it makes sense to think QM is not telling us there's something fundamentally strange about the universe that we couldn't have known otherwise.
If by "otherwise" you mean experimental observation of phenomona, for me QM certainly adds to the consternation. I can directly observe the duality of light, but then QM goes on to describe it in bizarre probablistic functions. I didn't know before QM that all light and matter exist in various degrees of indeterminism.

In fact, I suspect what QM may be doing is provide a formal proof of our complete inability to understand the world without making unjustified assumptions.
:biggrin: :rofl:
As far as I understand it, the whole of physics is a mathematical construct. I never believed those physical entities called laws, forces, fields, and so on, had any reality to them, I always thought of them as abstractions.
Well, certainly the "word" is not the "thing", but when you say "cat", that's also an abstraction, but you know cats exist. Ditto for gravity, magnetism, etc.
I still remember, in grade school, when our teacher told us that bodies in motion will remain in motion forever unless a force acts on them. I thought that couldn't be true as it went against our ordinary experience of motion, in which things always stop by themselves. He then explained that things stop because of "friction", and even as a child I immediately understood that sometimes you have to invent a fictitious entity to maintain the validity of a fictitious law. It's a wonder that it all works so well, but I still think of the whole structure of physics as nothing but a very clever fiction.
Umm, "friction" is no more fictitious than momentum. If I throw a brick at your head, will you duck, or assert the fiction of the situation? Abstractions are not the same thing as fictions.

It's not a problem of failed imagination, I see it as the result of a very powerful one. Just think what Newton would do if he thought as I did as a child: as soon as he came up with his laws of motion he would think, "nah, this doesn't make any sense unless I invent some additional entities to account for the cases when the laws don't hold, and that would not be right".
This wasn't the case in your example regarding momentum. Friction was not invented as a fudge factor. Friction is the "force that acts on them" that was specifically accounted for in Newton's law.

I do see you point, however, and dark matter and dark energy might turn out to be as fictitious as ether.
So physics is a big pile of laws that are just the product of human imaginations, plus a lot of purely imaginary entities that are required to account for the cases where the laws would otherwise not hold. I'm quite sure the photon is such an imaginary entity. (notice that I mean "imaginary" in the sense that friction is imaginary; the effects of an imaginary entity can be quite real if you assume their existence)
You don't seem to be distinguishing between abstractions like friction and photon, which describe observables, and things like string theory which is far removed from what we can feel confident about.
I hope you can have an idea from the above. Mostly, once we lost the ability to see what we are dealing with (the atomic level), all we were left with was the ability to make assumptions.
If you mean see with the naked eye, I think that's over-doing it. For example, I feel pretty confident that electron microscopes are actually telling us real things at the atomic level. There is a point though, where assumptions are being made which will eventually be overturned. That's integral to learning.

...
At a minimum you have to admit that science only explains the universe in general principles; in practice most problems are too complex to be approached scientifically.
Our current scientific explanation of the universe is certainly incomplete, but it is evolving. I'm wary of things that people say are "unknowable", or require "faith". It usually means that applying science would be stepping on a belief system.

I'd say science can be applied to problems of nearly any complexity. It's only when aspects of the problem are intangible (i.e. intergalactic space, additional dimensions, etc.) where science struggles most. I have no doubt that someday the mind will be fully understood, measured, modeled, even though the brain presents a hideous complexity. Consider the progress so far.
But that is only because you are confused. There is a perspective from which it is silly to worry about this stuff, except as an intellectual pastime. But here we're leaving the territory of physics and going into something far more important.

Ya, but as it turns out, this stuff affects my view of my existence as much as classical physics, evolution, neurology, etc. have shaped who I am. I've tried the "don't worry, be happy" approach, and I find it useful in bursts, but as unmaintainable as other theologies I've tried. You're right, of course, this begins to digress, but it does state why I care about QM, and its interpretations.
 
  • #39
Cane_Toad said:
If by "otherwise" you mean experimental observation of phenomona, for me QM certainly adds to the consternation. I can directly observe the duality of light, but then QM goes on to describe it in bizarre probablistic functions

My point was more along the lines that those microscopic phenomena have no bearing on our perception of the world. I do not believe QM describes the fundamentals of the universe, but that is mostly because I don't believe the universe has any fundamentals.

Friction was not invented as a fudge factor. Friction is the "force that acts on them" that was specifically accounted for in Newton's law.

I don't dispute that friction is a force, but I think the real reason why things move or don't move cannot be known because it doesn't exist. You can clearly see here how we think differently: you think friction must be real because you think the universe follows rules and friction is one of them; I think the universe has no rules and things move, or don't move, or sometimes move, or sometimes don't move, or any combination of that, for no particular reason.

It's just a different point of view, but it's a point of view that does not end up in some pretty serious dilemmas, which is why I prefer it.

I'd say science can be applied to problems of nearly any complexity.

Yes, but only in principle. In practice most problems are solved intuitively, often with better results than you'd get using a strictly scientific approach. In fact, the best thing about science is that it works together with our intuition -used on their own, each becomes a lot less powerful.

I have no doubt that someday the mind will be fully understood, measured, modeled, even though the brain presents a hideous complexity.

Sounds like a profession of faith to me :smile:

You may be right though. Maybe the brain is simpler than we realize. Then again, it may be more complex than we can possibly imagine.

Ya, but as it turns out, this stuff affects my view of my existence as much as classical physics, evolution, neurology, etc. have shaped who I am.

It shapes all of us, only in different ways. It's not easy living in modern times, there's just too much information, and most of it is wrong or irrelevant.

I've tried the "don't worry, be happy" approach, and I find it useful in bursts, but as unmaintainable as other theologies I've tried

What really worked for me was the realization that we are a mirror image of the universe, albeit in small scale. Sure the universe looks weird, but so do we and that is not reason for concern. For instance, whatever QM is doing to the universe it's also doing to you, and it's hard to believe it's a bad thing.
 
  • #40
I think the universe has no rules and things move, or don't move, or sometimes move, or sometimes don't move, or any combination of that, for no particular reason.

Only a naive realist would discuss whether or not the universe 'has' rules. We don't know anything about what the universe 'has', everything you have ever seen or experienced is a part of yourself.

If the question is "can humans ever formulate a system of rules that will be true in every case of motion?" then I get the feeling you would say no. But phrased in this way it is pretty clear that this view is a negative and not well founded impossibility claim, the sort of which histroy shows us have nearly always been wrong in the past.
 
  • #41
Crosson said:
Only a naive realist would discuss whether or not the universe 'has' rules.

I agree, except I wasn't discussing it. I said it was only a point of view.

If the question is "can humans ever formulate a system of rules that will be true in every case of motion?" then I get the feeling you would say no.

Actually, you would get a yes. For instance, I can offer the rule that "things will keep moving until they cease to move". It's not a very useful rule but seems absolutely true to me.

Now if you'd ask me "can humans ever formulate a system of rules that will be true in every case of motion without being essentially tautological", I'd say that is impossible as you'd have to live an eternity to assert the validity of the rules.

phrased in this way it is pretty clear that this view is a negative and not well founded impossibility claim, the sort of which history shows us have nearly always been wrong in the past.

So history has shown us that the universe is in fact governed by logical rules? I thought we were discussing precisely why QM seems to indicates the opposite :confused:

I detect two main approaches here: some people think QM doesn't make sense because the universe is screwed-up, while others think human knowledge will always break down and stop making sense at some point due to our inability to know everything. Call me naïve but I find the latter option far more plausible.
 
  • #42
Fra said:
A first question on a quick reading...

Why do you assume samples be statistically indepdendent?

/Fredrik


I guess for the same reason that the initial and final states are considered statistically independent. I labelled each state with a proposition whose truth or falsity are considered seperately from the truth or falsity of any of the others. This makes them "independent" from each other.

It doesn't seem possible to know the total number of states in the entire sample space of the universe since there are so many. So we can't know how probable anyone particular state is. And we can't know how probable it is to get an intersection of two sets of states, for example. Since there would be potentially an infinite number of total states in the entire sample space, each state would only have a differential absolute probability associated with it.

We can only know how probable it is to get a measured state given the initial state. Since we prepare the initial state before measurement, we know with absolute certainty that the initial state exists, so we give it a probability of one. Then given that we calculate how probable it is to propagate to measured states.

I made "implication" a square root of a probability because there were two "implications" for one "conjunction". But perhaps there is a more insightful reason for using the square root. Implication between two propositions can be expressed in terms of negation and disjunction, p->q = q+~p. The probability of the negation of p would be 1-P(p). And P(q+~p)=P(q)+P(~p). Now it occurs to me that when dealing with differential probabilities, this last expression may work out to something like (1-P(q)/2), which is the first approximation of the Taylor series for the square root of P(q). If someone would like to work out the details, I'd be grateful.
 
  • #43
nabuco said:
So history has shown us that the universe is in fact governed by logical rules? I thought we were discussing precisely why QM seems to indicates the opposite :confused:

So far it *has* been shown that the universe is governed by logical rules, within the domain they were forumulated. QM seems to be a new situation to me because it is a domain where the kind of logic[sensibility, really] we apply to the rest of the universe begins to break down.

One of the biggest problems I have with QM, is that our macro world is mostly governed by discrete objects and events, but in QM, non-locality seems to be the first consequence. Granted, in the macro universe, we do have fields and gravity, which aren't discrete (classically speaking), so have always been problematic. However, we humans have always been part of the discrete universe, and our ideas of ontology and epistomology have evolved accordingly.

QM (its math and phenomena) doesn't just pull the rug out from under all that, but it pulls the floor, foundation, and ground as well. Suddenly, we "know" nothing about the basis of reality. In our attempts to restore our understanding we are going to ever more extravagant lengths: specialized hyperdimensions, infinite universes, etc. It's a terrible time for Truth, in all senses of "terrible", good and bad. It's like acrophobia. Amazing new vistas too distant to see clearly, but our animal nature doesn't like the insecurity.
 
  • #44
Mike2 said:
I guess for the same reason that the initial and final states are considered statistically independent. I labelled each state with a proposition whose truth or falsity are considered seperately from the truth or falsity of any of the others. This makes them "independent" from each other.

I don't follow you. Given your assumption here they are independent that's clear, but I don't see what your motivation is for starting your reasoning with something that IMO a clear special case.

Are you trying to make some construct of underlying absolute probabilities to explain the relative probabilities?

Also, from what point of view does the reasoning take place? Who is in possession of the conjunction of all the facts of reality? and do they come from past experiments? If so, how long time do you need to collect all facts of reality?

I am not sure exactly I follow your thinking which is why I ask, but the premises doesn't make sense to me unless you can elaborate them.

/Fredrik
 
  • #45
Fra said:
I don't follow you. Given your assumption here they are independent that's clear, but I don't see what your motivation is for starting your reasoning with something that IMO a clear special case.

Are you trying to make some construct of underlying absolute probabilities to explain the relative probabilities?

Also, from what point of view does the reasoning take place? Who is in possession of the conjunction of all the facts of reality? and do they come from past experiments? If so, how long time do you need to collect all facts of reality?

I am not sure exactly I follow your thinking which is why I ask, but the premises doesn't make sense to me unless you can elaborate them.

/Fredrik

I started with the premise that all facts (whatever they are, whether we know them or not) are all consistent with each other - that it is never the case that one fact of reality proves false any other fact. Then I go from there to state the equivalence that all facts exist in conjunction. This all seems rather obvious. But it has implications. It implies that each fact is considered as a separate proposition from every other fact, or that facts are simple events, with only one sample per event, which are statistically independent from any other.

I'm not suggesting that I am in possession of all the facts. I am saying that whatever the facts may be, they can't contradict each other. This stipulation alone seems to lead to quantum mechanics. I am treating the facts in abstraction, and I'm not even trying to state for certain that some particular fact actually exists. That would require an observation to confirm. And we would not even know how many facts are exactly specified by an observation. It might be that my formulation still works even if each fact is an event of many subfacts as the samples within each event. I don't know.

Since we don't know exactly how many facts there are in the universe, we can never know by any means the absolute probability of anyone fact. We probably don't even know how to exactly define each fact to begin with. I only suspect that whatever the facts are, they can't contradict each other, and I submit this as a postulate from which to start. I don't even know what the relative probability is to obtain a second fact given the first. I only know that if a conjunction of facts leads to a multiplication by a probability, then implication must be the square root of a probability, since there are two implications for each conjunction.

I've tried to draw only from the realities of an abstract sample space, without resorting to numbers obtained from measurement. In this sense QM is derived from logic alone, as described at:

http://www.sirus.com/users/mjake/Physlogic.htm
 
Last edited by a moderator:
  • #46
Cane_Toad said:
So far it *has* been shown that the universe is governed by logical rules, within the domain they were forumulated.

I'm sorry but you cannot "show" that the universe is governed by logical rules, in any domain. Logical rules do not apply to reality, they only apply to our descriptions of it. What you really believe is that the universe can be described in a logical way, and with that I have no disagreement whatsoever. So much so that I'm positive there's something conceptually wrong with QM.

QM seems to be a new situation to me because it is a domain where the kind of logic[sensibility, really] we apply to the rest of the universe begins to break down.

Let me ask you something: apart from QM, can you think of any other situation in which you could be completely confounded by your own observations?

The best I can think of is watching a magician perform a clever trick. Good magicians can leave you dumbfounded, and the only reason you know they don't have the power to defy logic and commonsense is because you know such a thing is impossible.

As far as I can tell, what is happening in QM is that physicists discovered a trick not even they understand, and everyone is dumbfounded by what appears to be a violation of logic and commonsense. But you can only conclude or accept that if you think such a thing is possible. In essence, if you believe in magic.

In our attempts to restore our understanding we are going to ever more extravagant lengths: specialized hyperdimensions, infinite universes, etc.

Sure. Because "we put the rabbit in the hat and forgot about it" doesn't sound like a better explanation, right? I mean, how could we possibly do such a dumb thing and then act like idiots?

It's a terrible time for Truth, in all senses of "terrible", good and bad. It's like acrophobia.

I hope you don't take offense if I say I'm bemused by that comment. Science is not about truth, let alone "Truth". Science is about working models. I can understand your disappointment for not getting something out of science which you expected to be there, but I can only tell you that you were misled. To find Truth, you have to look beyond science (but you cannot ignore it)
 
  • #47
Mike2 said:
I started with the premise that all facts (whatever they are, whether we know them or not) are all consistent with each other - that it is never the case that one fact of reality proves false any other fact. Then I go from there to state the equivalence that all facts exist in conjunction. This all seems rather obvious. But it has implications. It implies that each fact is considered as a separate proposition from every other fact, or that facts are simple events, with only one sample per event, which are statistically independent from any other.

I'm not suggesting that I am in possession of all the facts. I am saying that whatever the facts may be, they can't contradict each other. This stipulation alone seems to lead to quantum mechanics. I am treating the facts in abstraction, and I'm not even trying to state for certain that some particular fact actually exists. That would require an observation to confirm. And we would not even know how many facts are exactly specified by an observation. It might be that my formulation still works even if each fact is an event of many subfacts as the samples within each event. I don't know.

Since we don't know exactly how many facts there are in the universe, we can never know by any means the absolute probability of anyone fact. We probably don't even know how to exactly define each fact to begin with. I only suspect that whatever the facts are, they can't contradict each other, and I submit this as a postulate from which to start. I don't even know what the relative probability is to obtain a second fact given the first. I only know that if a conjunction of facts leads to a multiplication by a probability, then implication must be the square root of a probability, since there are two implications for each conjunction.

I've tried to draw only from the realities of an abstract sample space, without resorting to numbers obtained from measurement. In this sense QM is derived from logic alone, as described at:

http://www.sirus.com/users/mjake/Physlogic.htm

Ok, regardless of the details of your paper I see to roughly speaking get what you are trying to do. Without getting into details I definitely agree that some parts of the QM formalism can consistently be thought of as a generalized probability theory, so in this sense I like your thinking, however there are some flaws in the ordinary QM. Some of these IMO has to do with the event space assumptions, and existence of a natural ordering of a sequence of event space partitions that seem necessary to recover the ordinary path integral. From a reality perspective, the assumption of some universal completely (in principle) known event space is not in my taste. To me it smells like a special case, which may happen to be significant enough to warrant study of course, but still.

I'm working on part of these things and hope that with time some of the fuzzy parts in the foundations should be cleared. I agree with you though that an analysis of the current formalism(which is admittedly flawed), may help pinpoint the exact position of the flaws.

/Fredrik
 
Last edited by a moderator:
  • #48
Science is not about truth, let alone "Truth". Science is about working models.

I hope you don't take offense if I say I'm bemused by that comment. You have shown yourself, Nabuco, as a logical positivist and a pyrrhonist. The realist vs positivist debate is an old one, and the leanings of the scientific majority majority towards one side or another tend to vary with time.

I present you stages of grief:

1) Pre-Awareness
2) Shock
3) Denial
4) Anger
5) Acceptance
6) Vehemence

The grief pertains to the individual having the first fundamental insight of philosophy: my eyes are not windows. Having gotten over his anger and denial, Nabuco strikes me as smugly comfortable in his acceptance stage. From this lofty point he shouts at the unenlightened:

I can understand your disappointment for not getting something out of science which you expected to be there, but I can only tell you that you were misled.

Acceptance is safe and comfortable, but ultimately it is weak, and vehemence is the only path to progress. The stages of grief do not apply only to individuals, they apply to period of western philosophy as a whole:

1) Pre-Socratics
2) Plato
3) Descartes
4) Soft Hume
5) Hard Hume
6) Post-Kantians

The final word on the sort of absolutes in question, universal and nonvacuous, remains to be written.
 
  • #49
nabuco said:
I'm sorry but you cannot "show" that the universe is governed by logical rules, in any domain. Logical rules do not apply to reality, they only apply to our descriptions of it.
...
I can't tell; are you talking about the subjective/objective reality question?

Secondly, I don't see how you can state with certainty that "logical rules do not apply to reality". If our models describe behavior (i.e. laws of motion) without fail, what is the point of gainsaying? Aside from whatever is going on with QM, what is your basis for saying that reality is not, in actuality, following rules that mirror our abstractions? At this point in history, that seems as presumptuous as saying "the universe obeys our rules".

"The word is not the thing", but if you're looking at a duck, and it's quacking back at you, you might as well concede that it's a "duck". That's a paraphrase of a paraphrase of Occam's Razor :smile:

But you can only conclude or accept that if you think such a thing is possible. In essence, if you believe in magic.
Yes, that's another way to put it. QM is showing us things that *are* magic. Historically, each time magic has been replaced by science, there has been an upheaval in our world view. So, my consternation with QM, is that it portends such an upheaval, and it is frustrating to be living on the crux (of unknown duration). And don't tell me to sit back and enjoy the ride. I am enjoying it, but at the same time, it's the shoe that refuses to drop, the Christmas present your mom has hidden.

It's a terrible time for Truth, in all senses of "terrible", good and bad. It's like acrophobia.
I hope you don't take offense if I say I'm bemused by that comment. Science is not about truth, let alone "Truth". Science is about working models. I can understand your disappointment for not getting something out of science which you expected to be there, but I can only tell you that you were misled. To find Truth, you have to look beyond science (but you cannot ignore it)

I didn't say science is about truth. However, truth comes from knowledge, and science is a major source, so, actually, I'm getting precisely what I expect from science. I was saying that Truth seems to be in flux (for the reasons above), and that can be frightening or exciting, because sometimes Truth must be shattered before being rebuilt.
 
  • #50
Cane_Toad said:
If our models describe behavior (i.e. laws of motion) without fail, what is the point of gainsaying?

Look, I think I'm misguiding you with this debate on whether the universe follows rules or not. Let's put that aside and agree that we are perfecly capable of coming up with a bunch of rules that correctly describe physical phenomena. I never said anything to the contrary although I understand why you think I did.

QM is showing us things that *are* magic.

Do you or do you not believe in magic, after all? If you don't, then you have to admit that QM needs to be reformulated in terms that keep the mathematical model and make more sense intuitively. And you have to believe such a thing is possible.

Historically, each time magic has been replaced by science, there has been an upheaval in our world view. So, my consternation with QM, is that it portends such an upheaval, and it is frustrating to be living on the crux (of unknown duration).

Ha ha ha! This time the upheaval went the wrong way, and the formerly skeptical physicists can now be caught talking about unbelievable quackery. Some fate :smile:

And don't tell me to sit back and enjoy the ride. I am enjoying it, but at the same time, it's the shoe that refuses to drop, the Christmas present your mom has hidden.

Interesting, I see your position now. You think we were on the verge of figuring out how things work, and suddenly we took a major step back. If that is the case, it's really bad.

I have never felt that physics allowed me to understand reality. Classical physics yes, it has had a profound impact on my understanding of the world, but the description of the atom never made any sense to me, and I doubt it ever made any sense to anyone. As far as I can tell it's just a working model of something too small to be observed, nothing else.

Truth seems to be in flux (for the reasons above), and that can be frightening or exciting, because sometimes Truth must be shattered before being rebuilt.

If it has to be shattered then it is not truth, it was illusion. Truth is eternal.
 
  • #51
nabuco said:
...
Ha ha ha! This time the upheaval went the wrong way, and the formerly skeptical physicists can now be caught talking about unbelievable quackery. Some fate :smile:
"This time"?! I'd say the irony of history is repeating itself. :smile: Seriously, though, do you have any examples? I don't know the personalities behind the various interpretations. I suspect that if Feynman were still around, that he'd be just as skeptical as ever about everything. My guess is that the hard core skeptics from pre-QM are still so, but the list is shrinking.

If you're talking in general terms about physicists, my second guess is that the distribution of skepticism hasn't changed much. What has changed greatly is the quantity. There are a lot more minds working today, so its natural to see more creative interpretations, especially since the problem has remained intractable for so long (accounting for the ratio of years / physicists ).

It may be a requirement of progress that wide-eyed new generations take the vanguard in each cycle. Consider Einstein's stubborn opposition (skepticism++ ) of QM. I've always wondered whether it came with age and success. It's sad that his flights of intuition weren't applied to QM.

Setting aside pseudoscience, the fascinating thing about a time like this, is that there is no way to tell whether the bizarre may bear out.

... but the description of the atom never made any sense to me, and I doubt it ever made any sense to anyone.
Ya, as soon as I heard about electron "clouds", it didn't taste right, but it was indistinguishable from other classical physics concepts taught in a watered down fashion. I.e. any two objects fall at the same rate in a vacuum. That seemed just as magic as anything else because nobody gave me the inertial explanation. Now if I'd asked "where does inertia come from?", I'd be right back to zero. Sadly (or suspiciously), the fundamentals of classical physics that are mysterious was never part of the curriculum.
If it has to be shattered then it is not truth, it was illusion. Truth is eternal.
Heh. There are precious few eternal truths outside of pure math and logic, maybe none, and fewer of those are useful in guiding lives and civilizations. It's much more useful to recognize eternal truth as a subset. We need more guidelines to direct ourselves. If we're smart, we try to base these truths on facts, but when the facts change, the mirror cracks (as soon as the Vatican gives permission, of course :smile: ).
 
  • #52
My thesis advisor clarified this issue quiet well to me:

There are two contexts in which we mean "interpretation". The scientific interpretation of a theory is the mapping between the formal language of the theory (mathematical objects) and the physical actions one performs in the laboratory/observatory.

The standard scientific interpretation of QM is well established:

(a.) Each Hilbert space vector (or rather the ray=spanned subspace of that vector) corresponds to some mode (method) of system preparation.

(b.) Each dual vector (ket) corresponds to some (possibly destructive) mode of final system registration.

(c.) The normalized transition amplitude:
[tex] A=\langle \phi\mid\psi\rangle [/tex]
has no direct meaning except that its square magnitude is the probability that a system initially prepared in the given mode [tex]\psi[/tex] will be finally registered in the given mode [tex]\phi[/tex]. (The Born Probability Interpretation!)

(d.) The Hermitian operators in the linear algebra over the Hilbert space correspond to acts of intermediate non-destructive measurement of some observable. The real eigen-values correspond to values which may be measured and the eigen-vectors correspond to the initial modes of preparation for which subsequent application of this measurement will yield with certainty the corresponding eigen-value. (Eigen-value pinciple!)
(This tells us how various modes of preparation/selection correspond to observable properties of the quantum.)

(e.) A system prepared in some initial mode and then for which a given measurement has been made is equivalent -in so far as any future behavior is concerned- to the system prepared in the mode corresponding to the eigen-vector obtained by projecting the initial mode vector onto the respective eigen-subspace of observable's Hermitian operator. (Mode projection principle which is poorly identified as "Wave function collapse")
Likewise with the dual sequence of observation and then final registration.

(One can then from (e.) and (c.) deduce the probabilities for sequences of observations.)

(f.) A system prepared in some prior mode corresponding to [tex]\psi[/tex] and the undergoing a specific dynamic evolution is equivalent to the system prepared in the mode corresponding to [tex]U\psi[/tex], the left product of a unitary operator in the linear algebra on the Hilbert space, said operator corresponding to the specific dynamic evolution. In particular one may construct this unitary operator by exponentiating the Hermitian observable corresponding to the measurement of the energy of the system in the usual way.
(One can then deduce probabilities for sequences of measurements given intermediate dynamic evolutions between the acts of observation.)

And that is all one needs to interpret QM in the context of scientific interpretation of a theory. All that is predicted is laid out in these points and in the mathematics. Any issue of interpretation beyond this leaves the realm of science (or at the very least deviates from QM).

What most people mean by "interpretation" is Metaphysical interpretation i.e. what is the reality of the system behind the phenomenology of measurements and actions in the lab/observatory.

The Copenhagen "Interpretation" in essence is the "Shut up and calculate" interpretation in that is doesn't attempt but rather denies the validity of attempting a Metaphysical interpretation. Quantum theory emphasizes observed properties as events rather than as conditions of reality. You see this in the above physical interpretation. Hilbert space vectors correspond to specific acts of system preparation. Hermitian operators correspond to specific acts of measurement.

You may object to this as just opinion but if you intend to actually carry out a specific quantum experiment it is the only interpretation you can possibly use.

You don't paint the "vertically polarized psi vector" onto the photons you paint it onto the device emitting those photons. You don't paint the Hermitian "left circularly polarized = +1, right circularly polarized = -1" matrix onto the photons you paint it onto the appropriate photon analyzer.

You can speculate all day about the reality of wave functions and the multiplicity of universes detached from our own. How is such speculation different from speculating about the gender of God or the multiplicity of blood types of unicorns?

Every physical act in a lab is expressed as a mathematical object or algebraic convention in the formal mathematical language of QM. The Born probability formula generates the predictions of the theory. The consequences of any experimental setup is already and completely predicted. That is to say QM doesn't predict any more than what the above six part interpretation implies. Any further "interpretation" is either...
(I.) a moot point scientifically speaking because it asserts facts about entities which cannot be empirically observed,
or
(II.) Not an interpretation at all but rather an extension or alternative theory as it is predicting more than or other than what is predicted already by QM.

This is what the Copenhagenists assert. People misrepresent the Copenhagen position by misstating that it asserts -say- "the moon doesn't exist when you aren't looking at it". This is not correct. Such an assertion is a positive statement about the reality of the moon and as such just as forbidden in the Copenhagenists position. Rather a proper statement would be:

It is improper in this quantum context to speak of the reality of the Moon one way or the other. We at most make predictions such as "If you look up on a clear night during certain days of the month you will see we call 'the moon'.".
In this context "the moon" is semantically an event of observation and not a Metaphysical object. It is not that one is denying the reality of the moon but one is speaking in a distinct language wherein "moon" or more usually "electron" is an empirical phenomenon rather than a metaphysical object.

Quantum theory in this context is then the epitome of pure science.
By science I mean: The epistemological discipline of empirically testable hypotheses.
It is separated from metaphysical speculation inherent in the language of objective reality. This is not to say in a social or philosophical context that "objective reality" is a bad concept. Rather in the specific discipline of science and more specifically in the theory of quantum phenomena it is "improper language" outside of the necessary classical description of the actual apparatus used to manipulate the system being described in the quantum theory.

Well that is my "rant" about questions of interpretation. I think the issues where clearly and fully resolved in Copenhagen nearly a century ago. Alternative interpretations are attempts to fit an actual square peg into an imaginary round hole. They are at best category errors and at worst gross mysticism.

It is the same type of problem that people who cannot recognize and excise their intuitive concept of absolute time have with special relativity which explicitly acknowledges that time as empirically defined is relative to which observers clock you mean.

Such individuals try to recast the predictions of SR in a non-relativistic context (i.e. invoking an aether). They don't predict any more or less than does Einstein but they feel more secure embedding the science within a mystical framework which preserves their cherished absolutes.

So too are the interpreting metaphysicists trying to embed the phenomenology of QM within some framework of objective reality beyond the observable. This is the very definition of mysticism.

Regards,
James Baugh
 
Last edited:
  • #53
jambaugh said:
My thesis adviser clarified this issue quiet well to me:
He did a good job with the formalism, and makes many points some people should hear to bring them down to earth, but he missed some things. Furthermore, such rigidity, as is seen in the application of the formalism [i.e. "blanket statement"], leads to complacent science. Let's continue.
There are two contexts in which we mean "interpretation". The scientific interpretation of a theory is the mapping between the formal language of the theory (mathematical objects) and the physical actions one performs in the laboratory/observatory.
I don't think anyone is going to mistake the non-"shut up and calc" interpretations listed in this thread as applicable to the laboratory, so I don't think this version of "interpretation" is the one under discussion.
...
What most people mean by "interpretation" is Metaphysical interpretation i.e. what is the reality of the system behind the phenomenology of measurements and actions in the lab/observatory.
Distinguish between "what *is* the reality", and "what *might be* [one] reality" behind the phenomenology. Besides, you put forth the above as a definition, but it's actually a [self-referential?] interpretation. "Interpretation" in the dictionary says nothing of the sort. Those are just two conventionalisms.

...
You may object to this as just opinion but if you intend to actually carry out a specific quantum experiment it is the only interpretation you can possibly use.
and here is what he missed: specific experiments are not the only way knowledge is advanced. Flights of fancy are often required to get the ball rolling. I think that's been pretty well established. As long as everybody acknowledges the speculative nature, everything's still scientific.
...
You can speculate all day about the reality of wave functions and the multiplicity of universes detached from our own. How is such speculation different from speculating about the gender of God or the multiplicity of blood types of unicorns?
You must distinguish been obvious lunacy, and speculations which might lead in the direction of new thought, hypotheses, and discovery.
Every physical act in a lab is expressed ... Any further "interpretation" is either...
"either/or" statements can be limiting, and can lead to arrogance.
...

Well that is my "rant" about questions of interpretation.
*Your* rant? You said it was your adviser's. You shouldn't adopt someone else's rant, it sounds like "hook, line, and sinker". Best to form your own misconceptions. :smile:
I think the issues where clearly and fully resolved in Copenhagen nearly a century ago. Alternative interpretations are attempts to fit an actual square peg into an imaginary round hole. They are at best category errors and at worst gross mysticism.
"A century ago". Doesn't just saying that fill you with a sense of stagnation? The Copenhagen interpretation of "shut up and calculate" is the only one that currently fits, and it would be great if more people understood that, but by it's nature means "stick on your blinders, and plow ahead". No progress has been made in understanding some of the very first phenomena, although they have been modeled to apparent perfection. A model which wraps a pragmatic set of equations around a contradiction isn't much of an explanation.
...
Such individuals try to recast the predictions of SR in a non-relativistic context (i.e. invoking an aether). ...
The difference is that SR does a good job of explaining "why" spacetime does weird things near C.
So too are the interpreting meta-physicists trying to embed the phenomenology of QM within some framework of objective reality beyond the observable.
That's silly, and flawed. Trying to fit the QM phenomenology into a framework which doesn't contradict all other observable phenomenology (in the sense that nothing above the QM level exhibits duality/non-locality/etc.) isn't "metaphysics". If you're going to say that there is nothing wrong with obvious contradictions between observables, then science would have stopped centuries ago.
This is the very definition of mysticism.

No,

mys·ti·cism (mst-szm)
n.
1.
a. Immediate consciousness of the transcendent or ultimate reality or God.
b. The experience of such communion as described by mystics.
2. A belief in the existence of realities beyond perceptual or intellectual apprehension that are central to being and directly accessible by subjective experience.
3. Vague, groundless speculation.

#2 says "belief". It's true that there is a lot of "belief" happening where it shouldn't, but there is also a lot of healthy skepticism, without throwing out the baby with the bath water.

#3 Many of these interpretations are far from "vague, groundless", though they are speculation.

All in all, your adviser's clarification makes no room outside what is the *practice* of QM, and a limited version of "interpretation" including only formalism and metaphysics.

There is a big hanging question, "What the hell does it all mean?". "Interpretation" seems like a good word to describe various attempts to answer this in a way that might also further the scientific model. "Channeling" is a good word for attempts such as marrying QM with consciousness. :smile:

in·ter·pret (n-tûrprt)
v. in·ter·pret·ed, in·ter·pret·ing, in·ter·prets
v.tr.
1. To explain the meaning of: interpreted the ambassador's remarks. See Synonyms at explain.
2. To conceive the significance of; construe: interpreted his smile to be an agreement; interpreted the open door as an invitation.
3. To present or conceptualize the meaning of by means of art or criticism.
4. To translate orally.

Most implications of "interpret" include a grain of salt, i.e. "that's just one interpretation".

What are we going to call the speculation, and though experiments and mathematical ideas required to form new directions in a field which has staggered to a halt to explain (not just model and predict it) the phenomena? Do you have a better suggestion other than "interpretations"? I'd suggest "speculations", but no self-respecting theorist is going to ruin his reputation working on "speculations". The problem is complicated by societal factors.
 
  • #54
Cane_Toad said:
He did a good job with the formalism, and makes many points some people should hear to bring them down to earth, but he missed some things. Furthermore, such rigidity, as is seen in the application of the formalism [i.e. "blanket statement"], leads to complacent science. Let's continue.
If the chapter is closed then such rigidity/blanket statement can goad the would be scientists back into the actual science and away from unanswerable debates.

A similar thing is needed in those forum debates which arise in the usenet because someone new has "discovered" a way to "explain/disprove" Einstein's theory in terms of some "aether theory" they've invented. The proper blanket statement, that relativity despenses with the need to invoke an aether rather than "disproves" an aether will when understood allow such debates to move on into actual questions of alternatives to SR.


I don't think anyone is going to mistake the non-"shut up and calc" interpretations listed in this thread as applicable to the laboratory, so I don't think this version of "interpretation" is the one under discussion.
You are overly optimistic. I've sat in discussions at a couple of quantum computation seminars where attendees have discussed building an instantaneous "Bell" telephone using Bhom's pilot waves to transmit info. Something which has been proved to be impossible.
Distinguish between "what *is* the reality", and "what *might be* [one] reality" behind the phenomenology. Besides, you put forth the above as a definition, but it's actually a [self-referential?] interpretation. "Interpretation" in the dictionary says nothing of the sort. Those are just two conventionalisms.
No I put it as stated, the other form of interpretation (and I paraphrased it for clearity). Again you can also include in such debates such silly notions that the reality behind the phenomena is a bunch of little angels talking to one another and moving particles about so that QM looks right. The question then is how to distinguish between various "realities". You can't know anything in a scientific context beyond the experimental verifications or invalidations of various physical predictions. The empirical epistimology of science does not address such questions or more precisely it specifically rejects such questions as outside its epistimological domain. Logic can only deduce self consistancy between various axioms, theorems and definitions. Faith (or asthetic choice of belief) can say anything but no two faiths are identical so a fixed group of people cannot use it to reach concensus.

What's more such hypotheses are of necessity "parochial" in that we tend to invent realities which conform to our evolved way of thinking (Our brains evolved to deal with a very narrow subdomain of physical phenomena i.e. the classical non-relativistic limit of a flat stationary Earth.).

Progress is more often made when we stop philosophically speculating about the nature of reality and pay attention to how and what we actually observe. I call attention to Einstein's insight when simply considered what an experimentalist means by "time". His answer "time is what a clock measures" allowed him to then realize "time is relative to which clock you are using".
He effectively "put blinders on" to the philosophical opinions about time and just paid attention to its pragmatic definition. This was what allowed him to better explain and better understand nature.

and here is what he missed: specific experiments are not the only way knowledge is advanced. Flights of fancy are often required to get the ball rolling. I think that's been pretty well established. As long as everybody acknowledges the speculative nature, everything's still scientific.
And I say that depends on the nature of said "flights of fancy". It is fine as in the case of Maxwell's mechanical model which helped him organize the relationship between the E and B fields. It is however a model which dictates the behavior of the empirical quantities. These speculations about the reality behind QM are not put forth as models but as interpretations. It is a very different thing and has a very different effect on "the progress of science".
You must distinguish been obvious lunacy, and speculations which might lead in the direction of new thought, hypotheses, and discovery.
That was what I thought I was doing here... as I see it e.g. Everetts many worlds is obvious lunacy.
"either/or" statements can be limiting, and can lead to arrogance.
and they can be correctly formulated thereby limiting misteps in future speculations. Arrogance is no crime. I think in fact a certain amount of arrogance in the form of audacity is necessary to embark on a career in theoretical physics. What is a crime is arrogant fallacy. So argue that I've left out a third alternative in my "either/or" or not but appealing to the asthetic value of humility is not in itself a counter argument.
*Your* rant? You said it was your adviser's. You shouldn't adopt someone else's rant, it sounds like "hook, line, and sinker". Best to form your own misconceptions. :smile:
Actually this is "my rant" I said he clearified it for me but this is my expression of my opinion which I acquired from his instruction. :wink:
"A century ago". Doesn't just saying that fill you with a sense of stagnation?
Yes. But maybe not in the same way for me as for you. I find the continued debate about "interpretation" tedious.
The Copenhagen interpretation of "shut up and calculate" is the only one that currently fits, and it would be great if more people understood that, but by it's nature means "stick on your blinders, and plow ahead".
You say "stick on your blinders, and plow ahead" I say
"don't get distracted by inconsequentials and plow ahead".

You say "toe mae toe" I say "toe mau toe".
No progress has been made in understanding some of the very first phenomena, although they have been modeled to apparent perfection. A model which wraps a pragmatic set of equations around a contradiction isn't much of an explanation. The difference is that SR does a good job of explaining "why" spacetime does weird things near C.
I disagree. The prior aether theory with Lorentz's transformations already explained the "why" namely travel through the aether slowed the clocks and shortened the measuring sticks. It was only when Einstein "put on blinders" and insisted on an operational interpretation of time and distance that he was able to excise the aetheric "reality" and indeed further generalize the theory to describe gravitation.

One problem there is that people mistakenly think GR says "gravity is just the curvature of space-time" i.e. "Gravity is just geometry". The proper interpretation is that we only know "geometry" by observing dynamics of particles and so there is no way to separate the two. It is that we formulate GR by observing that we can choose the geometry so that all of the dynamic gravitational forces disappear. And I further say understanding this very important distinction is critical to advancing the quantum gravitation program.
In fact I think it is one of the main reasons this program has stagnated.

That's silly, and flawed. Trying to fit the QM phenomenology into a framework which doesn't contradict all other observable phenomenology (in the sense that nothing above the QM level exhibits duality/non-locality/etc.) isn't "metaphysics". If you're going to say that there is nothing wrong with obvious contradictions between observables, then science would have stopped centuries ago.
I disagree with you that QM exhibits duality or non-locality. Born reciprocity is an artifact of the canonical treament of classical mechanics (if you are referring to duality between momentum and position). Wave-particle duality is related but the quantum theory doesn't pick out these two "wave" and "localized particle" as unique frames. We do in mapping various quantum phenomena back into our prior classical description. The Hilbert space for the electron is isotropic and no particular basis is preferred until you select a specific Hamiltonian, i.e. when you put the electron through various measuring devices. The only non-locality in QM occurs in descriptions not in the causal dynamics. The same occurs in classical physics and for the same reasons.
No,

mys·ti·cism (mst-szm)
n.
1.
a. Immediate consciousness of the transcendent or ultimate reality or God.
b. The experience of such communion as described by mystics.
2. A belief in the existence of realities beyond perceptual or intellectual apprehension that are central to being and directly accessible by subjective experience.
3. Vague, groundless speculation.

#2 says "belief". It's true that there is a lot of "belief" happening where it shouldn't, but there is also a lot of healthy skepticism, without throwing out the baby with the bath water.

#3 Many of these interpretations are far from "vague, groundless", though they are speculation.
Mysticism (ancient Greek mysticon = secret) ... In the context of epistemology, it can refer to using any kind of non-rational means — such as feeling or faith — in attempt to arrive at any kind of knowledge or belief.
ref: http://en.wikipedia.org/wiki/Mysticism"

In particular I am saying that e.g. belief in Everetts many worlds is a mystic belief. It cannot by definition be empirically verified and so must be taken on faith. Speculation about Everetts interpretation is no different in quality than speculating "why God let Daddy die". You can choose a belief that makes you happy but it isn't scientific in the least.
All in all, your adviser's clarification makes no room outside what is the *practice* of QM, and a limited version of "interpretation" including only formalism and metaphysics.

There is a big hanging question, "What the hell does it all mean?".
which beggs a question "in what context are we defining 'to mean' ". Which question is the one I attempted to answer with my post. There are the two contexts I gave and I assert that "scientific interpretation" is the empirical predictions alone. A theist may further ask your "What does it all mean" in the context of his theology. Making these metadistinctions is the whole point in using the qualifier "scientific".
"Interpretation" seems like a good word to describe various attempts to answer this in a way that might also further the scientific model. "Channeling" is a good word for attempts such as marrying QM with consciousness. :smile:

in·ter·pret (n-tûrprt)
v. in·ter·pret·ed, in·ter·pret·ing, in·ter·prets
v.tr.
1. To explain the meaning of: interpreted the ambassador's remarks. See Synonyms at explain.
2. To conceive the significance of; construe: interpreted his smile to be an agreement; interpreted the open door as an invitation.
3. To present or conceptualize the meaning of by means of art or criticism.
4. To translate orally.

Most implications of "interpret" include a grain of salt, i.e. "that's just one interpretation".

What are we going to call the speculation, and though experiments and mathematical ideas required to form new directions in a field which has staggered to a halt to explain (not just model and predict it) the phenomena? Do you have a better suggestion other than "interpretations"? I'd suggest "speculations", but no self-respecting theorist is going to ruin his reputation working on "speculations". The problem is complicated by societal factors.
If you are theorizing about as yet to be performed empirical observations then you call it a "speculative scientific theory". If you are hypothesizing about the nature of reality behind observed phenomena then there is already a very good word which is well defined in this context. It is called "a model". We speak of e.g. the quark model and the standard model.

Recall that the first question asked about quarks was "Why then aren't isolated quarks observable?" Until and unless they are (even if only indirectly) the quark model remains a "model". At the point where we actually see hadron jets comming from high energy nucleon collisions we have begun promoting the term "quark" from a mere model element to an actual physical phenomenon. But the chapter is not quite closed on this. It is instructive to look at the footnotes on the various properties of quarks in the CRC handbook. They qualify what quantities are based on theories.

It is conceivable that an alternative model may better predict subnuclear phenomena. But contrast this with fundamentally invisible Bhom pilot waves and fundamentally invisible alternative Everett universes which say nothing about the empirical predictions which isn't said already. This is why they are not called models. They come "after the fact" of the theory's formulation.

The term model as distinct from theory is well hashed out in science and in particular physics. Models are useful (often damned useful) in acting as a scaffolding to conceptually organize the components of a theory.

The distinction is however ignored by too many physicists. One example awhich makes me cringe is the topic of "string theory" which is rather a class of "string models". The problem I think is that "theory" in mathematics very often juxtiposes with "model" in physics. I suppose "string theory" is short for "the mathematical theory of string models" et al.

And I agree with you that if treated as a model which might extend the predictions of theory beyond QM then these "interpretations" may be of value.
But the intent behind their formulation is not this. Rather they are --by their nature-- attempts to "take a step backward" and recast quantum phenomena in a manifestly classical fundament.

When viewed in abstract the phenomenological language of quantum theory is a richer than the ontological language of classical theory. You can always revert a quantum theory to a classical one by fixing upon a specific basis and commuting subalgebra of observables. You of course don't get the same classical system as the one which was quantized to form the quantum system. Rather you can always embed a given classically description of a physical system inside another QM system as a particular commuting subset of the quantum observables and system modes.

Said another way, quantum theory rejects certain implicit assumptions built into classical ontologically based physics. Specifically the assumption that a single ontological model will describe all phenomena. It is thus a more general scope in which to describe physical phenomena. Any attempt to re-embed this phenomenological language within an ontological one is regressive and leads to such absurdities as infinite universes or pilot waves which causally interact backward through time.

Reality is a state of mind. Our brain updates the reality model in our heads continuously to conform to the sensory phenomena we experience. It does such a good job of it (on the scale of our direct experience) that it is easy to (almost impossible not to) mistake the mental objects for the phenomena they model. Everett's many worlds is actually perfectly correct [ul]IF[/ul]
you understand those "worlds" to be conceptual objects and not real except in this unreal redefinition of "reality" I'm giving.

Regards,
James Baugh
 
Last edited by a moderator:
  • #55
Another thought with regard to decoherence (part of the description of one of the poll items). I started a thread "Is the entropy of the universe zero" wherein I suggested we may view the entropy of a system as a measure of its entanglement with its environment.

In this sense then we can say that decoherence is the process of this environmental entanglment occurring.

Regards,
James Baugh
 
  • #56
jambaugh said:
If the chapter is closed then such rigidity/blanket statement can goad the would be scientists back into the actual science and away from unanswerable debates.

Eeek. I hope you really don't mean that. That's a recipe for bitterness all around.

...
because someone new has "discovered" a way to "explain/disprove" Einstein's theory in terms of some "aether theory"
...
attendees have discussed building an instantaneous "Bell" telephone using Bhom's pilot waves to transmit info.
...

There is no good answer to the problem of idiots. If you try to make the idiots behave by limiting imagination, you'll still have the idiots, but the now you've repressed the talented.

...
Progress is more often made when we stop philosophically speculating about the nature of reality and pay attention to how and what we actually observe.

A common fallacy. Both are required, and more. I seem to remember most of the greats saying that intuition was an integral part of their process, even though the scientific process was the ostensible method.

I call attention to Einstein's insight when simply considered what an experimentalist means by "time". His answer "time is what a clock measures" allowed him to then realize "time is relative to which clock you are using".
He effectively "put blinders on" to the philosophical opinions about time and just paid attention to its pragmatic definition. This was what allowed him to better explain and better understand nature.

To look beyond prevailing attitudes is the opposite of blinders. I think his success was much more complicated that than. Setting aside questions of inate talent, if all he ever knew was "time is what a clock measures", and that's all anybody ever bothered to find out, why would he even think/care to go further? It was partly because all the many voices had made the world a complex enough place to generate the knowledge of a need.

I'll submit that once he started dogmatically sticking to relativity, and rejecting "spooky science" - his definition of poor science, i.e. QM, the pinnacle of empiricism, this is when his career effectively ground to a halt.

And I say that depends on the nature of said "flights of fancy". It is fine as in the case of Maxwell's mechanical model which helped him organize the relationship between the E and B fields. It is however a model which dictates the behavior of the empirical quantities. These speculations about the reality behind QM are not put forth as models but as interpretations. It is a very different thing and has a very different effect on "the progress of science".

Agreed.

However, it's not the word "interpretation" that is the real problem. It's the people ignorant of the difference between speculation, theory, etc., and the system that let's them get through grad school this way [high school too, maybe even preschool].

... as I see it e.g. Everetts many worlds is obvious lunacy.

Arrogance is no crime. I think in fact a certain amount of arrogance in the form of audacity is necessary to embark on a career in theoretical physics.

Arrogance and audacity are very different things, with very different effects on other people. Equating them is to give yourself license for bad behavior.

What is a crime is arrogant fallacy.

Yes, but fallacy isn't. It's a natural part of the human process. Some tolerance must be made for the bright-eyed and bushy tailed.

Stubborn loyalty to fallacy isn't a crime either, just a genetic defect which hopefully will be self-correcting.

So argue that I've left out a third alternative in my "either/or" or not but appealing to the asthetic value of humility is not in itself a counter argument.

A third alternative is always "something you haven't thought of yet". You must not close your mind because other minds are too loose.

Are you really saying you think there is no area between pragmatism and mysticism?

Yes. But maybe not in the same way for me as for you. I find the continued debate about "interpretation" tedious.

The debates can be annoying, but silence is worse. This is the reason we prise freedom of speech. We *MUST* allow stupidity to have it's voice; history has shown us the alternative.

I disagree. The prior aether theory with Lorentz's transformations already explained the "why" namely travel through the aether slowed the clocks and shortened the measuring sticks.

"why" wasn't the big failure with aether.

I disagree with you that QM exhibits duality or non-locality. Born reciprocity is an artifact of the canonical treament of classical mechanics (if you are referring to duality between momentum and position). Wave-particle duality is related but the quantum theory doesn't pick out these two "wave" and "localized particle" as unique frames.
...The only non-locality in QM occurs in descriptions not in the causal dynamics.

What you just saw happen in front of your eyes didn't really happen. QM doesn't distinguish them as distinct, so you didn't see it.

Or, maybe you're saying that duality/etc. is accounted for in the math, and that the QM framework doesn't have these paradoxes. This is no surprise, since QM was formulated to deal with them.

Perhaps I wasn't clear. I was saying that the phenomena that lead to QM exhibit these problems.

I'm not an expert, so I'm wondering if I've got it all wrong. I haven't done the experiments myself, but I'm relying on what's been reported. Does wave-particle duality phenomena exist? What exactly do you call the behavior seen in entangled particles? It smells like non-locality.

[The only non-locality in QM occurs in descriptions not in the causal dynamics. ]
The same occurs in classical physics and for the same reasons.

I don't follow the part about classical.

...

In particular I am saying that e.g. belief in ...

Yes! Belief and faith are words antithetical in meaning to science, IMHO. Whether the belief is in something proven or foolish only clouds the issue. Nobody should be believing anthing in science, but constantly questioning.

which beggs a question "in what context are we defining 'to mean' ". Which question is the one I attempted to answer with my post. There are the two contexts I gave and I assert that "scientific interpretation" is the empirical predictions alone. ...

I especially object to black and white thinking in this case. When we ask, "what does it all mean?", we should be asking in as many contexts as possible. And then it is critical to remember which answer came from which context.

... "speculative scientific theory". ... It is called "a model".

Better terms all. Unfortunately, history shows us that language bows to the entrenched masses. Also, remember not to throw out the baby with the bath water because you don't like the word "interpretation", a child doesn't choose its parents. It might be a speculation that leads to the next great thing, as you continue with below:

And I agree with you that if treated as a model which might extend the predictions of theory beyond QM then these "interpretations" may be of value.
But the intent behind their formulation is not this.

Yeh, but the intent behind a new idea can be forgiven (not forgotten). New ideas are always welcome as long people are educated about the hazards.

Rather they are --by their nature-- attempts to "take a step backward" and recast quantum phenomena in a manifestly classical fundament.

I still don't understand why you think that looking for a bridge is bad. Since QM doesn't work much in the classical domain yet, classical is still important. Until QM works everywhere, trying to do such a recast has potential value.

I think you make a good point that there *might* be no useful intersection between QM and CM, and more people should know this.

Also, trying to extend the context of understanding for QM does not mean only "classlcal", and I don't see that all the "interpretations" won't have results for QM, even if the original motivation was for CM. Using the CM fundiment as a model for our seach is a starting point, not a move backwards.

What is it you're proposing instead of looking for bridges between QM and CM?

When viewed in abstract the phenomenological language of quantum theory is a richer than the ontological language of classical theory. You can always revert a quantum theory to a classical one by fixing upon a specific basis and commuting subalgebra of observables.

I'll have to take your word for it. It makes me wonder about decoherence.
Isn't it supposed to do this, but is still in progress?

...

Said another way, quantum theory rejects certain implicit assumptions built into classical ontologically based physics. Specifically the assumption that a single ontological model will describe all phenomena. It is thus a more general scope in which to describe physical phenomena.

Hmm. Where then is the drive for consistency? Once you have a model that tests well, why try for a better one? If later something turns out not to fit into it, just tack on another one. Maybe there isn't a single model, but I'm not ready to accept that it's time to throw out the idea.

Any attempt to re-embed this phenomenological language within an ontological one is regressive and leads to such absurdities...

This has happened, yes. But I don't see the case for "any" attempt. My hope is that QM language will inform CM and ontological frameworks.

I also wouldn't label anything absurd that extends logically from QM, and which has no clear contradictions with fact. Sure, many worlds might be untestable, but it's not stupid either, and you can't rule out that an offshoot won't be testable. "Absurd" is a denigration which should be reserved for ideas such as using HUP to exert control over the world through the power of wishing.

In general, you give too little latitude to that which is outside the scope of strict QM, and perhaps too much latitude to what's inside.

I also feel that there is a growing possibility that there will be no unifying sense to be had. I'm also having a general problem with fatalism. I wouldn't want to discourage others from "audacity", though.

Lastly, the thing that concerns me most is your tone is so strong. One feels a crusade coming on. :smile: Science needs people like you to provide a balance, but if you don't try to meet the "interpretationists" half way, it's going to be just one more head butting session leaving everybody further entrenched.
 
  • #57
The real one. Voting won't change reality.
 
  • #58
Cane_Toad said:
Eeek. I hope you really don't mean that. That's a recipe for bitterness all around.
Why would you say so? I'd give examples in my own learning about QM but the exposition gets rather involved.

Don't get me wrong. I've nothing against speculation about parallel universes and new forms of waves interacting. I've similarly no objections to an aether type theory. What I get all upset about is false declarations e.g. about what Quantum Theory actually says which are in fact declarations of what these "interpretations" say. And likewise about what SR says w.r.t. aether.
To look beyond prevailing attitudes is the opposite of blinders.
A false dichotomy. The third alternative is to examine prevailing attitudes, understand their basis and "separate the wheat from the chaff".

Not all "prevailing attitudes" are without merit and sometimes (and I assert it in this case) the prevailing attitude is the best one. But more to the point:
It is fine to look beyond the theory of QM and speculate and let your imagination run free. But don't clump such speculation as part of quantum theory and most especially don't clump non-operational philosophical speculations in with scientific theories.
However, it's not the word "interpretation" that is the real problem. It's the people ignorant of the difference between speculation, theory, etc., and the system that let's them get through grad school this way [high school too, maybe even preschool].
I disagree. Words mean things and the word "interpretation" use outside the operational interpretation I first gave means specifically stepping outside the epistemological discipline that science relies on. In this sense a "scientific ontological interpretation" is an oxymoron. Only in the classical domain where one can be both "scientific" and work in an ontological language can the two words be compatible.
Are you really saying you think there is no area between pragmatism and mysticism?
In the epistemology of belief, yes I so say. However one pragmatic necessity is to recognize when we don't have the time or energy to test every hypothesis with an experiment. In so doing we admit ignorance. Indeed ignorance is empirically demonstrable. In the arena of beliefs we are either pragmatic of mystical. In the arena of actions we pragmatically acknowledge the limits of our knowledge and act tentatively on insufficient data. Some, especially prosthletizing theists trying to justify their faith will claim tentative action is a kind of faith e.g. "you have faith that the bridge you are crossing won't fall down". Rather such "faith" is either belief backed by some empirical data or trust based on the need to act in the absence of data. Trust here being an action in the absence of knowledge rather than knowledge in the absence of empirical evidence!
The debates can be annoying, but silence is worse.
...can be... but when the debate by its nature resolves nothing then silence is golden!
This is the reason we prise freedom of speech. We *MUST* allow stupidity to have it's voice; history has shown us the alternative.
Prizing freedom of speech is different from prizing the speech. Diversity of opinion for diversity's sake is stupid. Diversity is a neutral quality and not inherently virtuous. Remember that "Freedom of Speech" is freedom from violent coercion to silence. It is the absence of that coercion which is prized and not the diversity of speech for diversity's sake.
"why" wasn't the big failure with aether.
No. The aether explanations only failed in failing to be necessary. Einstein succeeded in recognizing this fact and using that insight to generalize velocity relativity to include gravitational accelerations.
What you just saw happen in front of your eyes didn't really happen. QM doesn't distinguish them as distinct, so you didn't see it.
Huh? I'm saying stick to saying what happened as "something happened" and stop supposing a particular form of objective reality lies behind it...at least as a foundation. Mind you tentative reality models are not just "ok" but essential to functioning in the practical scale. It is rather important to understand that such ontological description is a conceptual framework resting on top of a foundation of phenomenology... specifically when you are dealing with foundational issues such as physical theories. In effect "ontological objects" are phenomena satisfying certain criteria, said criteria not necessarily always satisfied when we push the limits of empirical observation close to hbar.
Or, maybe you're saying that duality/etc. is accounted for in the math, and that the QM framework doesn't have these paradoxes. This is no surprise, since QM was formulated to deal with them.

Perhaps I wasn't clear. I was saying that the phenomena that lead to QM exhibit these problems.

I'm not an expert, so I'm wondering if I've got it all wrong. I haven't done the experiments myself, but I'm relying on what's been reported. Does wave-particle duality phenomena exist? What exactly do you call the behavior seen in entangled particles? It smells like non-locality.
That's because you are smelling with your "interpretation" nose. Once you excise the idea that the wave functions are referring to real objects and accept rather that they are a description of possible empirical determinations about the physical system then the non-locality ceases to be a physical non-local action and becomes nothing different than similar classical examples.

The expected value of all but one lottery ticket suddenly jump to zero when the winning ticket is selected. This isn't a strange action at a distance but rather a property of "expected value". When you understand the expectation value to be a relationship between physical properties and not itself a physical property then its sudden non-local change is no mystery.
I don't follow the part about classical.
Waves are classical physical systems. Particles are classical physical systems.

Electrons as described in quantum theory are phenomenological processes which are neither wave nor particle simply because "wave" and "particle" are as stated classical objects.

We none-the-less may actualize certain measurements of an electron which also correspond to measurements of classical particles, as too with certain wave-esque measurements. These are just two of a continuum of ways we may resolve a maximal set of observables for an electronic quantum. Each such resolution yields a description which we may then identify with some classical object "solitons, clouds, wavelets" most of which we don't have a sensible physical analogue to but which none-the-less we could describe classically.
Yes! Belief and faith are words antithetical in meaning to science, IMHO. Whether the belief is in something proven or foolish only clouds the issue. Nobody should be believing anthing in science, but constantly questioning.
Careful there! "Belief" and "Faith" are in different categories.
Belief is the whole point both of science and of faith.

We should believe the empirical outcomes of experiments as scientists, just as the "faithful" should believe they are going to heaven when. Each belief follows from the posited epistemological discipline and that sets the value system i.e. it puts the "should" in should believe.

Faith and science are antithetical to each other by definition as they are distinct epistemologies. But belief is the essential coin of both of these kingdoms.

I especially object to black and white thinking in this case. When we ask, "what does it all mean?", we should be asking in as many contexts as possible. And then it is critical to remember which answer came from which context.
You can't make make a silk purse out of a sow's ear nor can you find any virtue in the act considering the possibility for diversities sake. Similarly you can't make ontological speculations about unobservable entities fit inside the qualifier "scientific". Some issues are black and white and painting them gray is destroying the meaning of what they invoke.
Better terms all. Unfortunately, history shows us that language bows to the entrenched masses.
But that is true only of common language. Technical language historically does the opposite. If you read the history of science and mathematics you will see much of the improvement in the field comes about by systematic discussion about and removal of ambiguities in the language.
Also, remember not to throw out the baby with the bath water because you don't like the word "interpretation", a child doesn't choose its parents.
I love the word "interpretation"...so much so that I defend it against the abuses to which it is being subjected. The word already has a specific meaning in the context of science: "interpretation=operational interpretation" . And again I assert that the ontological interpretations here discussed are necessarily extra-scientific.
It might be a speculation that leads to the next great thing, as you continue with below:
. . .
Yeh, but the intent behind a new idea can be forgiven (not forgotten). New ideas are always welcome as long people are educated about the hazards.
Of such hazards I here attempt education.
I still don't understand why you think that looking for a bridge is bad.
Since QM doesn't work much in the classical domain yet, classical is still important. Until QM works everywhere, trying to do such a recast has potential value.
Some "bridges" are also called "gangplanks" and must be raised if the ship is fulfill its purpose. We are now trying for example to generate a quantum theory of gravitation. I assert we need to stop keeping the S.S. Quantum Cruiser tied to the classical home dock. Cast off the "canonical quantization of a classical model" ropes and see what she can discover when allowed to sail past the horizon.

An ontological foundation is not necessary. Requiring one must therefore limit the range of possible theories. More importantly quantum theory was founded on ignoring ontology and paying attention to operational meaning. That gave it the power to predict beyond the domain of classical theories. Why hamstring any further extension to QM by insisting on ontological interpretation?

Lastly, the thing that concerns me most is your tone is so strong. One feels a crusade coming on. :smile: Science needs people like you to provide a balance, but if you don't try to meet the "interpretationists" half way, it's going to be just one more head butting session leaving everybody further entrenched.

Well when "the heathens are pillaging the holy places" sometimes a crusade is just the thing!:biggrin:

Yes my tone is strong. (Although sometimes it is easy to read stronger tone in a text forum than is intended) It is as I see it a vital issue in making true headway rather than the current research scenario of "well I don't know where we are but we're making good time!"

Well I think I've said all I can in this thread without becoming repetitive. So I'll leave you the last word (which I will read but likely not reply to).

Regards,
James Baugh
 
  • #59
This gives me a much better picture of what you've been saying. Your effort is appreciated.

jambaugh said:
I disagree. Words mean things and the word "interpretation" use outside the operational interpretation...

I think we saying nearly the same thing, and I'll concede that "interpretation" has been shanghai'd.

I think the reason for this is because of the educational process, though. It's a tough thing: teaching mental rigor without mental ossification.

Remember that "Freedom of Speech" is freedom from violent coercion to silence. It is the absence of that coercion which is prized and not the diversity of speech for diversity's sake.

Agreed, though I have a preference for thoughtful diversity for its own sake, but at the same time I wish everybody spoke the same language. I guess I prefer thoughtful hypocrasy too.

I was pointing out that you have to be ready to accept diversity as a consequence. One must let the child babble that he might learn.

Once you excise the idea that the wave functions are referring to real objects and accept rather that they are a description of possible empirical determinations about the physical system then the non-locality ceases to be a physical non-local action and becomes nothing different than similar classical examples.

I understand that the paradoxes don't really exist inside the QM framework.

My problem is that I still want to fix the ontological framework, since that's where I live, and QM is the only game in town. :smile:

Careful there! "Belief" and "Faith" are in different categories.
Belief is the whole point both of science and of faith.

We should believe the empirical outcomes of experiments as scientists,

This might be a personal thing. I've come to the conclusion that almost all my failings in science and life can be traced to belief in things I was lead to believe were true and factual. I now endeavor, with varying degrees of success, to assign probabilities of certainty where ever I recognize I have a belief.

I assert we need to stop keeping the S.S. Quantum Cruiser tied to the classical home dock.

Ok, fine. Just tell us where you go and what you find, and we'll try to live with it. :rolleyes:

An ontological foundation is not necessary. Requiring one must therefore limit the range of possible theories. More importantly quantum theory was founded on ignoring ontology and paying attention to operational meaning. That gave it the power to predict beyond the domain of classical theories. Why hamstring any further extension to QM by insisting on ontological interpretation?

Like I said, I want to improve the place I live in, but I can see the argument to sever the rope. However, I'd like to replace it with a wireless, so QM can keep everybody informed. I think there is a need for trying to resolve the ontological paradoxes, but I see how it should be a separate discipline. It does seem to be a monkey wrench for QM.

Well I think I've said all I can in this thread without becoming repetitive. So I'll leave you the last word (which I will read but likely not reply to).

Regards,
James Baugh

I have one more question, if you would. I'm in the spot where I'm seeing the foundation of reality as we knew it turning transparent under foot, and looking down through the glass, all I can be sure of is that once again (sigh) I'm going to have to accept that everything I know is wrong. Well, that's overstating it, but that's how it feels.

How would you counsel us, oh Seer of the Pure QM, to integrate the nature of light, the substructure of probability, existence through formulation, into our happy little lives? We can see through the looking glass, darkly, but we can't step through yet. As our sight grows keener, the shore recedes.
 
  • #60
Cane_Toad said:
This gives me a much better picture of what you've been saying. Your effort is appreciated.
...
I think we saying nearly the same thing, and I'll concede that "interpretation" has been shanghai'd.
Yes, I think we were arguing around each other. I believe the main issue is one of explicitly "putting on the physicist's hat" and explicitly "taking it off". That way one knows to use narrower definitions of terms. Its similar to the formality of a court proceeding, or when the mathematician adopts a specific axiomatic system and definition set.
I understand that the paradoxes don't really exist inside the QM framework.
Then you're way ahead of the game! Very good!
My problem is that I still want to fix the ontological framework, since that's where I live, and QM is the only game in town. :smile:
Examine that desire... it isn't a bad one per se. It simply stems from the practical utility of ontological frameworks when they are possible i.e. when you work (as is usually the case) within a classical domain. As an exercise translate a given ontological framework into an empirical process language. Then you will begin seeing when it is and is not appropriate to utilize one. Further it may help to see when working in an ontological framework limits one's perspective.

Take for example the issue of quality control in various institutions. If you are managing an automotive plant then you must have an objective model of your product. Your quality control methods measure deviation from this standard and you reject or remediate products which fail.
Now consider the case of educating children. The degree of education of a child does not conform to an ontological "state of mind". Children are processes and not objects. Attempts to work with an ontological framework (exemplified by most state imposed "quality based education" standards) results in teaching students to be better at taking tests. In order to truly educate the child you must train and then trust the teacher. The teacher then educates by establishing a personal relationship with the child, encourages the child's mind to grow in different ways.

Yes you must have some objective standards but they are not based on an ontological model of the children's minds. Creativity and imagination, two of the most important qualities of an educated child, can be recognized and encourage but they can never measured in terms of conformity to an ontological model.
This might be a personal thing. I've come to the conclusion that almost all my failings in science and life can be traced to belief in things I was lead to believe were true and factual. I now endeavor, with varying degrees of success, to assign probabilities of certainty where ever I recognize I have a belief.
Try viewing the quest for knowledge as itself a process of discovery and self growth (of society and of individuals) instead of simply the task of topographic record keeping. I have a sign over my desk which reads:
All knowledge is a form of self knowledge.
It is something of an exaggeration but emphasizes a point that all "facts" are of necessity relative to both the perspective of the observer and to the structure of the language in which it is expressed. That the sun revolves around the Earth is not false in this context. But it should be understood as a description of our dynamic relationship to the sun and not a truth in an absolute framework.
Like I said, I want to improve the place I live in, but I can see the argument to sever the rope. However, I'd like to replace it with a wireless, so QM can keep everybody informed. I think there is a need for trying to resolve the ontological paradoxes, but I see how it should be a separate discipline. It does seem to be a monkey wrench for QM.
"seem to" begin the operative phrase. I see it as QM pointing out that fixating on ontology is the monkey wrench. But you are right in the need for the ability to translate back to the layman's domain of experience. I think strong analogies of the correct type are helpful.

One I often use is polling people. Clearly the order in which questions are asked matters w.r.t. how they are answered. Clearly being "a republican" vs "democrat" is really a question of how one votes i.e. behaves and not a "state of mind". Clearly when you ask someone a question they have been changed. Thus people are similar to quanta in that they are process like phenomena and not clockworks describable in terms of objective ontological states. The threshold is one of sensitivity to the observational process when one examines behavior. How someone actually will behave in a set of circumstances is not directly measurable by simply asking them. You must put them to the acid test. Likewise how an electron will behave in all circumstances is not encoded in an intrinsic state and can only be defined by and in terms of acts of measurement.
I have one more question, if you would. I'm in the spot where I'm seeing the foundation of reality as we knew it turning transparent under foot, and looking down through the glass, all I can be sure of is that once again (sigh) I'm going to have to accept that everything I know is wrong. Well, that's overstating it, but that's how it feels.
When studying very intently on my graduate research I reached a point where this seemed to be the case. The analogy I felt was that of wading into deeper water until I could no longer feel firm ground beneath my feet. I realized I was reaching that point where I would have to learn to swim... but also that I could drown (in total relativity) if I wasn't careful.

Pushing the analogy further, I think the thing to do is construct a boat (e.g. a tentative ontological framework which is understood as such and with clearly defined connection to the fluid realm of phenomenological experience) said boat fits in with our prior analogy as well, able to leave the homeland and seek out new worlds of understanding while keeping us from drowning in the nihilism of total amorphous relativity.

Remember that good ole classical Newtonian mechanics is right in its domain. We shouldn't worry about relativistic effects as we hammer nails and we needn't worry about how examining the hammer's position too closely will necessitate uncertainties in its momentum. It is not that the world is crumbling. It is that as we gain more perspective we see what we first thought of as the whole world is just an island in a much larger sea.

This is the curse of growing. We loose the certainties of youth. We learn our parents are mortal and that our home towns are not the whole universe.
How would you counsel us, oh Seer of the Pure QM, to integrate the nature of light, the substructure of probability, existence through formulation, into our happy little lives? We can see through the looking glass, darkly, but we can't step through yet. As our sight grows keener, the shore recedes.

Meditate my son upon the operational meaning of these concepts.

The nature of light is how it behaves. ("there is no spoon!")

Probabilities are normalized frequencies of events. Look to the nature of those events for structure or lack thereof.

Existence exists, formulation is something we do.

As our sight grows keener it is not we who recede from the shore? Turn around and face forward.

and finally when contemplating reality remember the immortal words from Monty Python's Quest for the Holy Grail... "It's only a model!":biggrin:

Regards,
James Baugh
 
  • #61
I pick "Shut up and calculate!" I'm more in agreement with Nabuco, don't sweat the small stuff like metaphysics. QM works marvelously while metaphysics is as useless as a politician. If you can't accept that all I can suggest is get over it!

I would guess that even when we combine relativity and qm there will still be at least two uniquely useful ways of interpreting the theory. Personally I'm rooting for noncommutative fractal geometry and utter chaos. They seem to be about as perfectly opposite as it gets, yet, leave lots of room for every conceivable interpretation.
 
  • #62
Sorry for the gravedig, but there's something that I need to say.

jambaugh, you say that any QM needs to be freed of the 'expectations' of a classical theory.
I assert we need to stop keeping the S.S. Quantum Cruiser tied to the classical home dock. Cast off the "canonical quantization of a classical model" ropes and see what she can discover when allowed to sail past the horizon.
However, from what I've heard of the Copenhagen Interpretation(CI), I don't think that it truly does that. The reason I think this is that CI asserts that measurement apparati/observer is classical. This necessitates a line between quantum and classical behaviour. This line (I've heard it being called the Heisenberg cut) has been the cause of much grief over QM, with such ill-defined concepts such as macroscopicity and consciousness being proposed as the location of the cut. Also, CI, while of great use in calculating what happens in quantum systems, implies that QM is incomplete, because CI explicitly denies the universality of QM.
 
  • #63
Aeroflech said:
Sorry for the gravedig, but there's something that I need to say.

jambaugh, you say that any QM needs to be freed of the 'expectations' of a classical theory.

However, from what I've heard of the Copenhagen Interpretation(CI), I don't think that it truly does that. The reason I think this is that CI asserts that measurement apparati/observer is classical. This necessitates a line between quantum and classical behaviour. This line (I've heard it being called the Heisenberg cut) has been the cause of much grief over QM, with such ill-defined concepts such as macroscopicity and consciousness being proposed as the location of the cut. Also, CI, while of great use in calculating what happens in quantum systems, implies that QM is incomplete, because CI explicitly denies the universality of QM.

This cut is a cut in description not in the actuality. We don't need to write our theorems or record our data on "quantum paper" (or rather we don't need to take the paper's quantum nature into account).

In the CI quantum theory is "quantum complete" in that it is a maximal description. You must distinguish ontological completeness which classical theory claims (and is its failing) with empirical completeness which quantum theory under CI (or any other "interpretation") claims. Indeed it is the fact that all "interpretations" of QM are indistinguishable empirically which shows that as a matter of science "interpertation" is a theological question. CI is the most "atheistic".

None of this invalidates my point which is one about heuristics and not about interpretation. In our attempts to generate new/better theories we follow the old route of quantizing a classical description of a system. Naturally we historically converted the older classical descriptions to quantum ones as we moved into the new paradigm. But I think we've just about gone as far as we can there and its time to lay down that habit and think in terms of generating quantum theories and classicalizing them to see whether they correspond to the current remaining purely classical ones e.g. gravity.

This has essentially already been done in the quantum field theories of gauge forces in the standard model. There was never a classical treatment of the strong and weak interactions beyond some rough square well nuclear models. And yet the heuristic was still followed in the formulation. The standard model gauge theories were constructed first as classical gauge fields and quantized. Though the classical model wasn't taken seriously the same old formula was followed. It was of course very successful in so far as it went...however...

I assert that the problem with field theories in general and similarly with string theories is that they begin with a classical space-time description and preserve it throughout. String/brane theory abstracts this a bit with its multidimensional manifolds in a fixed higher dimensional spaces but its still fundamentally a quantization scheme applied to an underlying classical model.

The actuality of it is that the universe is fundamentally quantum. At some scales a classical descripiton is sufficient and we may easily explain how quantum systems may in the large scale, with restrictions on observables, behave classically. But there is no reason to suppose that there exists out there a classical model of some imagined universe which via a quantization scheme will generate the correct description of ours. Even if such is true it doesn't seem to be the most efficient way to go about it.

And this isn't a purely academic point. The decision on what research to fund is crucial and if the channeling of funds into a single branch motivates researchers to give up alternatives then we'd better be consicous of which avenues are dead ends and which are more direct paths to the better theory.
 
  • #64
Many Worlds Interpretation

Some proponents of MWI hold that other worlds except the actual world had better be understood as merely "possible worlds" or possibilities rather than real, parallel worlds. If MWI is read this way, it is not metaphysically queer at all. Are there any problems with this (revised) "Many Worlds Interpretation"? Please let me know. Thanks.
 
  • #65
metaethics said:
Some proponents of MWI hold that other worlds except the actual world had better be understood as merely "possible worlds" or possibilities rather than real, parallel worlds. If MWI is read this way, it is not metaphysically queer at all. Are there any problems with this (revised) "Many Worlds Interpretation"? Please let me know. Thanks.

If anything this is even more metaphysically bizarre. The "worlds" in MWI are severely constrained. Normally philosophers take anything not logically impossible to be possible. But MWI cannot do this. For example, it is not logically impossible for the laws of quantum mechanics to be completely wrong!

So your interpretation needs to justify the constraints placed on possibility in these constrained-possible-worlds, explain their ontology (Do we imagine them? Do they really exist in some non-physical realm?), and explain why one of these constrained-possible-worlds is actualised while the others are not.
 
Last edited:

Similar threads

  • Quantum Physics
Replies
13
Views
620
Replies
2
Views
802
Replies
44
Views
3K
  • Quantum Interpretations and Foundations
Replies
29
Views
1K
  • Sticky
  • Quantum Physics
Replies
1
Views
5K
Replies
11
Views
1K
  • Quantum Physics
Replies
6
Views
1K
  • Quantum Physics
Replies
3
Views
281
Replies
10
Views
2K
Back
Top